LSAT and Law School Admissions Forum

Get expert LSAT preparation and law school admissions advice from PowerScore Test Preparation.

User avatar
 Dave Killoran
PowerScore Staff
  • PowerScore Staff
  • Posts: 5852
  • Joined: Mar 25, 2011
|
#85536
Complete Question Explanation
(The complete setup for this game can be found here: lsat/viewtopic.php?f=307&t=9348)

The correct answer choice is (D)

Answer choice (A) is incorrect because it violates the third rule.

Answer choice (B) is incorrect because it violates the first rule.

Answer choice (C) is incorrect because it violates the second rule.

Answer choice (D) is the correct answer.

Answer choice (E) is incorrect because it violates the fourth rule.
User avatar
 emerald archer
  • Posts: 1
  • Joined: Apr 01, 2021
|
#86022
Apply one rule at a time and eliminate wrong answers until you see the last man standing.

A) G S J L V Z
B) G V Z J L S
C) L J G V Z S
D) L S G J Z V
E) Z G J L S V

If it's a hotel, it isn't inspected on Wednesday. The hotels are G, J, and L. Answer B violates the rule by placing L on Wed morning. B is gone. G comes before J. Answer C violates the rule and is gone. G and S are in an anti block, meaning they can't share the same day. Answer A violates the rule and is gone. If Z is inspected in the morning so is L, E places Z in the morning yet leaves L in the afternoon. It violates the rule and Answer E is gone.

D is correct.
 Administrator
PowerScore Staff
  • PowerScore Staff
  • Posts: 8917
  • Joined: Feb 02, 2011
|
#86023
Hello Emerald,

Thank you for your post and for pointing out this error! You are correct! D is the correct answer.

It has been corrected and my apologies for the mistake :).

Thanks again!

Julie Lipscomb
Director of Tutoring and Admissions
PowerScore Test Preparation
1-800-545-1750

Get the most out of your LSAT Prep Plus subscription.

Analyze and track your performance with our Testing and Analytics Package.